prep U patho Exam 4

Pataasin ang iyong marka sa homework at exams ngayon gamit ang Quizwiz!

What can the nurse assume about a child's behavior when faced with the need to repeat a painful procedure? Pain causes similar responses in people regardless of their age. A child will benefit from a matter-of-fact approach on the part of the nurse. Children act to avoid pain based on their memory of past painful events. A child will resist any intervention that involves contact with his or her body.

Children act to avoid pain based on their memory of past painful events. Explanation: Children do feel pain and have been shown to reliably and accurately report pain. They also remember pain. This is evidenced in studies of children with cancer, whose distress during painful procedures increases over time without intervention, and in neonates in intensive care units, who demonstrate protective withdrawal responses to a heel stick after repeated episodes. The other options may not necessarily be true of most children.

The nurse is assessing a client with acute pancreatitis who denies use of alcohol. When teaching the client about additional causes of acute pancreatitis, the nurse includes which of these in the discussion? Select all that apply. Increased cholesterol Hypocalcemia Gallstones Potassium-sparing diuretics Abdominal trauma

Gallstones Increased cholesterol Abdominal trauma

A client is asked to stand with feet together, eyes open, and hands by the sides. Then the client is asked to close the eyes while the nurse observes for a full minute. What assessment is the nurse performing? Segmental reflex Crossed-extensor reflex Proprioception Posture

Proprioception

After several months of persistent heartburn, an adult client has been diagnosed with gastroesophageal reflux disease (GERD). Which treatment regimen is likely to be prescribed for this clients GERD? Proton pump inhibitors; avoiding large meals; remaining upright after meals Antiinflammatory medications; avoiding positions that exacerbate reflux; a soft-textured diet Surgical correction of the incompetent pylorus and limiting physical exercise Weight loss and administration of calcium channel blocking medications

Proton pump inhibitors; avoiding large meals; remaining upright after meals

A nurse is teaching a client diagnosed with Crohn disease about potential complications. The most appropriate information for the nurse to include would be: Chronic constipation Difficulty swallowing Fistula formation Excessive weight gain

Fistula formation

Crohn's disease has a distinguishing pattern in the gastrointestinal (GI) tract. The surface has granulomatous lesions surrounded by normal-appearing mucosal tissue. A complication of the pattern includes which of the following? Dysphagia Rectal bleeding Constipation Fistula formation

Fistula formation

Which types of cells are supporting cells of the peripheral nervous system? Ependymal cells Astrocytes Schwann cells Oligodendrocytes

Schwann cells Explanation: The Schwann cells play an important role in supporting the peripheral nervous system. The other cells support the central nervous system.

The nurse has just completed an assessment on a client admitted with Guillain-Barré syndrome. The nurse determines that a priority of care will be: feeding the client. ventilatory assessment and support. emotional support. administration of antibiotics.

ventilatory assessment and support.

The patient who has experienced third-degree burns is susceptible to which of the following specific types of gastrointestinal (GI) ulceration? Duodenal Gastric Stress Peptic

Stress

The nurse taking a report on a client coming into the emergency room plans care for a client with brain dysfunction based on which symptom? Wheezing Chest pain Pupils that react to light Stupor

Stupor The most frequent sign of brain dysfunction is an altered level of consciousness such as stupor. Pupils that react to light, wheezing, and chest pain are not symptoms of brain function.

The MRA scan of a client with a suspected stroke reports ruptured berry aneurysm. The nurse plans care for a client with: Thrombotic stroke Encephalitis Lacunar infarct Subarachnoid hemorrhage

Subarachnoid hemorrhage The rupture of a berry aneurysm leads to a subarachnoid hemorrhage.

A client is admitted to the hospital with a suspected diagnosis of strangulated bowel. The nurse anticipates the client will need: Surgery to release the bowel Lower abdominal massage Low fiber diet for 24 hours Insertion of a nasogastric tube

Surgery to release the bowel

Neurons communicate with each other through which structure? Neural crest cells Cell bodies Synapses Dendrites

Synapses Explanation: Neurons communicate with each other through structures known as synapses. Cell bodies and dendrites are covered in synapses. Neural crest cells migrate aways from the forming neural tube and are progenitors to the parasympathetic nervous system.

A nurse assessing an older adult 3 hours postoperative notes guarding, grimacing, and stiff body movement when changing positions. The client denies feeling pain. Which response will help the nurse best manage this client's pain? "You seem to be uncomfortable. Pain is common with surgery. I can bring you pain medication." "You just had surgery. Are you sure you are not having any pain?" "Are you afraid of becoming addicted if I give you any pain medication?" "I can come back in an hour to see if you are having any pain then."

"You seem to be uncomfortable. Pain is common with surgery. I can bring you pain medication." Explanation: Older adults are often reluctant to report pain if it might be a bother to others, or they assume they should experience pain as a result of increased age and health issues. The nurse needs to be specific in asking questions about the pain an older adult is experiencing while using therapeutic communication skills. A closed-ended question will not provide as much data as more open-ended responses will.

Reflex activity involves which neurons? Interneurons Afferent neurons Efferent neurons All of the above

All of the above

The nurse working in an emergency room is caring for a client who is exhibiting signs and symptoms of a stroke. What does the nurse anticipate that the physician's orders will include? Intravenous antibiotics CT scan Pain medication MRI

CT scan The nurse should anticipate that the client will be ordered a CT scan to rule out hemorrhagic stroke that would preclude the administration of tissue plasminogen activator (tPA).

The nurse contacts the health care provider regarding a client's early signs of diminishing level of consciousness based on which manifestations? Select all that apply. Stupor Disorientation Blunted responsiveness Inattention

Disorientation Blunted responsiveness Inattention The earliest signs of diminution in level of consciousness are inattention, mild confusion, disorientation, and blunted responsiveness.

Which neuron connects sensory and motor neurons? Cranial nerves Interneurons Afferent neurons Efferent neurons

Interneurons

Select the correct structural sequence of the GI tract.

Mouth, esophagus, stomach, duodenum, jejunum, ileum, cecum, colon, rectum

The nurse is preparing a client for oculovestibular reflex assessment (cold caloric test). The nurse explains that the test is used to elicit which of the following? Nystagmus Decorticate posture Pupillary light response Tinnitus

Nystagmus The oculovestibular reflex test (cold caloric test) consists of instilling cold water into the ear canal to elicit nystagmus.

A client is experiencing chest pain that radiates to the left arm and neck. The nurse would interpret this pain as: Somatic Referred Cutaneous Visceral

Referred Referred pain is pain that is perceived at a site different from its point of origin but innervated by the same spinal segment. -Visceral pain originates in the visceral organs and is one of the most common pains produced by disease, -cutaneous pain arises from superficial structures, and - somatic pain originates in deep body structures

A client reports chronic cramping, bloating and diarrhea and is diagnosed with a deficiency in brush border enzymes within his small intestine. Which meal/snack is high in carbs and protein and likely will exacerbate the client's signs and symptoms? Roast beef and a baked potato. Tossed salad with an oil and vinegar dressing. Tortilla chips and guacamole. Grapefruit and prunes.

Roast beef and a baked potato.

The results of a patient's 24-hour stool specimen indicate 20 g or more of fat. The nurse would interpret this as: Steatorrhea Clostridium difficile Diarrhea Hyperlipidemia

Steatorrhea

A nurse at a long-term care facility provides care for a client who has had recent transient ischemic attacks (TIAs). What significance should the nurse attach to the client's TIAs? TIAs are relatively benign phenomena that necessitate monitoring, but not treatment. TIAs, by definition, resolve rapidly, but they constitute an increased risk for stroke. The small bleeds that define TIAs can be a warning sign of an impending stroke. TIAs result in an accumulation of small deficits that may eventually equal the effects of a CV.

TIAs, by definition, resolve rapidly, but they constitute an increased risk for stroke. TIAs can be considered a warning sign for future strokes. They are not hemorrhagic in nature and their effects are not normally cumulative. They may require treatment medically or surgically.

Which of the following statements is true concerning gastric enterocytes? They secrete enzymes that aid in digestion of proteins. They digest entero bacteria. They provide binding sites for insulin. They secrete lubrication for the gastrointestinal tract.

They secrete enzymes that aid in digestion of proteins.

Ion channels in nervous system cells generate action potentials in the cells. What are the ion channels guarded by? Schwann cells Satellite cells Ligand gates Voltage-dependent gates

Voltage-dependent gates Explanation: These membrane channels are guarded by voltage-dependent gates that open and close with changes in the membrane potential.

The nurse walks into a room and finds the patient forcefully expelling stomach contents into a wash basin. When documenting this occurrence, the nurse will use the term: Retching. Expatriate. Vomiting. Nauseous.

Vomiting.

A client has been recently diagnosed with Alzheimer disease and has been prescribed a cholinesterase inhibitor. This drug will slow the progression of the client's symptoms by: increasing the frequency of action potentials. increasing the rate of action potentials. decreasing ACh breakdown. decreasing ACh synthesis.

decreasing ACh breakdown. Explanation: Acetylcholine is secreted by the cholinergic nerve endings and is rapidly broken down by the enzyme acetylcholinesterase. The cholinesterase inhibitor inhibits the breakdown of the neurotransmitter in the synaptic space, thereby increasing the effect of ACh. ACh levels do not affect action potentials in presynaptic or postsynaptic membranes.

Which statement best describes the pathophysiology of Parkinson disease? degeneration of the nigrostriatal dopamine system increase in the number of dopamine receptors by an alternate basal ganglion failure of the cerebral cortex, which does not allow the use of acetylcholine rise in acetylcholine levels, causing an inhibition of voluntary movement

degeneration of the nigrostriatal dopamine system

The swallowing reflex is an entirely voluntary activity. False True

false

The nurse is aware that the primary function of the sympathetic nervous system is: maintenance of organ function during periods of minimal activity. maintenance of vital functions and responding when there is a critical threat to the integrity of the individual. suppression of responses during threatened periods. conservation of energy and resource replenishment.

maintenance of vital functions and responding when there is a critical threat to the integrity of the individual. Explanation: The sympathetic division maintains vital functions and responds when there is a critical threat to the integrity of the individual—the "fight-or-flight" response. The parasympathetic nervous system is concerned with conservation of energy, resource replenishment, and maintenance of organ function during periods of minimal activity.

What will conduct injurious stimuli to alert the body to potential damage?

nociceptors Explanation: Nociceptors are sensitive to painful and noxious stimuli and alert the system to injury. Thermoreceptors will perceive heat, proprioceptors will perceive body position, and odorant receptors will perceive the sensation of smell.

After being thrown off the back of a bull, a bull rider can move his arms but has loss of motor function in the lumbar and sacral segments of the spinal cord. This is referred to as: Anterior cord syndrome Quadriplegia Paraplegia Tetraplegia

paraplegia

Nystagmus due to cerebellar dysfunction would most likely interfere with which activity? Walking Reading Fine motor skills speech

reading Explanation: Conjugate readjustment of eye position due to cerebellar damage can make reading very difficult.

During a flu shot clinic, one of the questions the student nurse asks relates to whether the client has a history of Guillain-Barré syndrome. The client asks, "What is that?" How should the nursing student reply? "A type of paralysis that affects movement on both sides of the body. It may even involve the respiratory muscles." "A degenerative disease where you have trouble walking without the help of a cane or walker." "Swelling of your arm where you got your flu shot and maybe your eyes and lips had some swelling as well." "Influenza-like illness where you had fever and chills for 2 to 3 days after your last flu shot."

"A type of paralysis that affects movement on both sides of the body. It may even involve the respiratory muscles."

The unique clinical presentation of a 3 month-old infant in the emergency department leads the care team to suspect botulism. Which assessment question posed to the parents is likely to be most useful in the differential diagnosis? "Is there any mold in your home that you know of?" "Is there any family history of neuromuscular diseases?" "Have you ever given your child any honey or honey-containing products?" "Has your baby ever been directly exposed to any chemical cleaning products?"

"Have you ever given your child any honey or honey-containing products?"

A client is being taught how to use a TENS unit .The nurse determines that teaching was effective when the client states: "Anytime that the setting on my TENS unit needs to be changed I will have to return it to the company and wait for them to deliver a new one." "I will need to return to the doctor's office to have this TENS unit surgically implanted under my skin for the best pain relief." "I should take my medication and apply the TENS unit to the painful area as soon as I feel the pain." "The TENS unit is the easiest and most cost-effective way to manage my pain at home."

"I should take my medication and apply the TENS unit to the painful area as soon as I feel the pain." Explanation: Early intervention for pain relief is the best practice. The client should understand that he or she should take his or her pain medication and apply the TENS unit. TENS units are noninvasive and should be applied as soon as the client starts to perceive pain. It is often used in conjunction with pain-relieving medication.

A student is comparing the two patterns of contractions in the small intestine. Which of the following statements is most accurate? "Peristaltic movements begin in the cecum." "Segmentation waves function mainly to mix the chyme with the digestive enzymes from the pancreas." "Peristaltic movements drive the contents forward and backward." "The frequency of segmenting activity decreases after a meal."

"Segmentation waves function mainly to mix the chyme with the digestive enzymes from the pancreas."

A beta-adrenergic blocker has been prescribed for a client diagnosed with migraines. The most important information for the nurse to teach the client would be: "Take the medication only when feeling a migraine start." "Stop this medication immediately if you have dizziness after taking it." "This medication will prevent you from having any more migraines." "Take the medication daily as you have been directed."

"Take the medication daily as you have been directed." Explanation: Preventive medications such as beta-adrenergic blockers should be taken consistently to prevent vascular changes from occurring. In most cases, preventive treatment must be taken daily for months to years. They should not be stopped abruptly and should be weaned or tapered off.

The spouse of a client diagnosed with Alzheimer disease asks the nurse why the client often neglects to take a shower. The spouse states that the client was always diligent with hygiene in the past; however, over the past few months that has not been the case. What is the nurse's best response? "The client would be fine without showering." "The client is experiencing a temporary relapse." "The client just does not care anymore." "You should remind the client to shower."

"You should remind the patient to shower." The client should be reminded to shower because most likely he or she has difficulty remembering to do so. In the moderate stage of Alzheimer disease, which can last for several years, it is not unusual for hygiene to be neglected because the person may just not remember if he or she showered. There is no information in the question to support the remaining responses.

The nurse is reviewing the medical history of four clients. The nurse determines that the client at greatest risk for developing peptic ulcer disease is: A client diagnosed with arthritis who takes acetaminophen (Tylenol) twice per day A client with a prior diagnosis of Helicobacter pylori and refused treatment A client who overeats excessive amounts of fatty foods A client who has a history of a ruptured appendix

A client with a prior diagnosis of Helicobacter pylori and refused treatment

A 26 year-old female is resting after a 1-minute episode during which she lost consciousness while her muscles contracted and extremities extended. This was followed by rhythmic contraction and relaxation of her extremities. On regaining consciousness, she found herself to have been incontinent of urine. What has the woman most likely experienced? A complex partial seizure An absence seizure A tonic-clonic seizure A myoclonic seizure

A tonic-clonic seizure. A tonic-clonic seizure often begins with tonic contraction of the muscles with extension of the extremities and immediate loss of consciousness. Incontinence of bladder and bowel is common. Cyanosis may occur from contraction of airway and respiratory muscles. The tonic phase is followed by the clonic phase, which involves rhythmic bilateral contraction and relaxation of the extremities.

A nurse caring for a patient with an intestinal obstruction anticipates which of the following assessment findings? Select all that apply. Abdominal distension Vomiting Diarrhea Abdominal pain

Abdominal distension Vomiting Abdominal pain

The nurse is caring for a client admitted to the emergency room with suspected meningitis. The nurse prepares to perform which nursing intervention upon physician orders, while diagnostic testing is being completed? Administration of oxygen Administration of antibiotics Adminstration of pain medication Administration of TPN

Administration of antibiotics The nurse should prepare to administer antibiotics as ordered by the physician while the diagnostic tests are being completed. Delay in initiation of antimicrobial therapy, most frequently due to medical imaging prior to lumbar puncture or transfer to another medical facility, can result in poor client outcomes.

For seizure disorders that do not respond to anticonvulsant medications, the option for surgical treatment exists. What is removed in the most common surgery for seizure disorders? Entorhinal cortex Temporal neocortex Hippocampus Amygdala

Amygdala The most common surgery consists of removal of the amygdala and an anterior part of the hippocampus and entorhinal cortex, as well as a small part of the temporal pole, leaving the lateral temporal neocortex intact. Only a portion of the hippocampus and entorhinal cortex, and temporal pole are removed.

When assessing the client with acute pancreatitis, which of these diagnostic tests, consistent with the disease, does the nurse anticipate will be altered? Glucose values Creatine kinase Amylase and lipase The transaminases

Amylase and lipase

A nurse is teaching a client newly diagnosed with a seizure disorder. Which statement is most important for the nurse to provide regarding antiepileptic medications? Children can build up a tolerance to the medication quickly. All antiepileptic medications should be taken with food. Pregnant women should reduce the dose of medication or discontinue until after delivery. Antiepileptic medications should never be discontinued abruptly.

Antiepileptic medications should never be discontinued abruptly. Consistency in taking seizure medications is essential to obtaining and maintaining therapeutic blood levels of the medication. Abrupt withdrawal can cause seizure recurrence. Monitoring and assessment of drug levels are important. Each prescribed drug will provide information regarding administration to provide client safety.

An elderly patient presents with loose mucousy stools. The nurse suspects the patient has Clostridium difficile. What is a priority assessment for the nurse? Ask the patient about his or her fluid intake. Ask the patient about his or her normal bowel pattern. Ask the patient about the foods he or she has consumed. Ask the patient about his or her antibiotic use.

Ask the patient about his or her antibiotic use.

An elderly patient presents with loose mucousy stools. The nurse suspects the patient has Clostridium difficile. What is a priority assessment for the nurse? Ask the patient about the foods he or she has consumed. Ask the patient about his or her normal bowel pattern. Ask the patient about his or her antibiotic use. Ask the patient about his or her fluid intake.

Ask the patient about his or her antibiotic use.

A patient has experienced a stroke affecting the reticular formation of the medulla and lower pons. The nurse tells the patient's wife that care must be taken with eating to prevent which of the following? Duodenal ulcers Gastric atony Aspiration pneumonia Steatorrhea

Aspiration pneumonia

The nurse caring for a client with an aneurysmal subarachnoid hemorrhage understands that the most common cause of this condition is: Berry aneurysm Subdural hematoma Hypertension Arteriovenous malformation

Berry aneurysm Aneurysmal subarachnoid hemorrhage is a type of hemorrhagic stroke caused by the rupture of a cerebral aneurysm and resultant bleeding into the subarachnoid space. Most of these aneurysms are small, saccular aneurysms called berry aneurysms.

A client has sustained a severe, diffuse brain injury that resulted in seriously compromised brain function. The client is at greatest risk for: Confusion Brain death Amnesia Paraplegia

Brain death Severe brain injury that results in seriously compromised brain function can result in brain death.

When the suspected diagnosis is bacterial meningitis, what assessment techniques can assist in determining if meningeal irritation is present? Brudzinski sign and Kernig sign Brudzinski sign and Chadwick sign Kernig sign and Chadwick sign Chvostek sign and Guedel sign

Brudzinski sign and Kernig sign Two assessment techniques can help determine whether meningeal irritation is present. Kernig sign is resistance to extension of the knee while the person is lying with the hip flexed at a right angle. Brudzinski sign is elicited when flexion of the neck induces flexion of the hip and knee.

Which statement best conveys an aspect of the role of cerebrospinal fluid (CSF)? CSF distributes plasma proteins throughout the superficial gray matter of the CNS. CSF cushions the brain and provides a near-water medium for diffusion of nutrients. It provides physical protection for the brain and ensures leukocytes and erythrocytes are evenly distributed in the CNS. It ensures the high metabolic and oxygenation needs of the brain are met, as well as absorbing physical shocks.

CSF cushions the brain and provides a near-water medium for diffusion of nutrients. Explanation: In addition to providing a cushion for the CNS, CSF provides a medium that is 99% water, in which nutrients, electrolytes, and wastes can be diffused. It is not centrally involved in the distribution of oxygen, plasma proteins, or blood cells.

Which of the following statements is true concerning the digestion of carbohydrates? Carbohydrates require hydrochloric acid to be absorbed. Carbohydrates are broken down into monosaccharides before being absorbed. Carbohydrates must be digested in the large intestines. Carbohydrates are excreted without being digested.

Carbohydrates are broken down into monosaccharides before being absorbed.

A student is studying gallbladder function. Which of the following gastrointestinal hormones stimulates contraction of the gallbladder? Gastrin Ghrelin Cholecystokinin Secretin

Cholecystokinin

Cerebrospinal fluid (CSF) is produced by which structure? Dura mater Arachnoid mater Falx cerebri Choroid plexus

Choroid plexus Explanation: The CSF is produced by tiny, reddish masses of specialized ependymal cells and capillaries, called the choroid plexus, which projects into the ventricles. The other options do not produce CSF.

All diseases have risk factors. What is the most significant environmental risk factor for pancreatic cancer? Water pollution Cigarette smoking Heavy metal toxicity Air pollution

Cigarette smoking

A severe type of headache that occurs more frequently in men than women and is described as having unrelenting, unilateral pain located most frequently in the orbit is called: Migraine headache Tension headache Cluster headache Chronic daily headache

Cluster headache Explanation: Cluster headache is a type of primary neurovascular headache that typically includes severe, unrelenting, unilateral pain located, in order of decreasing frequency, in the orbital, retro-orbital, temporal, supraorbital, and infraorbital region.

The client asks the health care provider, "What is the purpose of applying cold to a sprained ankle?" Which response by the health care provider is best? Cold causes extravasation of blood into the damaged area. Cold provides alternating vasoconstriction and dilatation when applied locally. Cold provides pain relief and suppresses the release of products from tissue damage. Cold dilates the blood vessels and can be left on for as long as needed.

Cold provides pain relief and suppresses the release of products from tissue damage. Explanation: Cold prevents the release of products of tissue damage and provides pain relief (produces vasoconstriction at the site). Application of heat would cause blood vessels to dilate and more blood to be drawn to the local area.

An elderly patient has been placed on a broad-spectrum antibiotic for a recurrent urinary tract infection. Which of the following potential problems would the nurse anticipate in this patient? Dehydration Inability to eat Transmission of Escherichia coli Colonization of Clostridium difficile

Colonization of Clostridium difficile

An older adult client expresses concern about straining to pass stools that are small and hard. What term with the health care provider document in this client's chart? Irritable bowel syndrome Diverticulosis Dehydration Constipation

Constipation

The thick area of myelinated axons that connects the two sides of the cerebral cortex is known as which component? Globus pallidus Corpus callosum Caudate nucleus Striatum

Corpus callosum Explanation: The thick area of myelinated axons that connects the two sides of the cerebral cortex is known as the corpus callosum. The remaining options are all components of the basal ganglia.

A client suffering global cerebral ischemia a week after a suicide attempt by hanging is in the intensive care unit receiving treatment. The parent asks the nurse why it is necessary to keep the client paralyzed with medications and on the ventilator. The most appropriate response would be that these therapies: decrease the client's ability to attempt suicide again. decrease metabolic needs and increase oxygenation. increase oxygen demands and metabolic needs. decrease intracranial fluid volumes and pressures.

Decrease metabolic needs and increase oxygenation The general goal of treatment with global cerebral ischemia is to decrease metabolic needs and increase oxygenation to the injured cerebral tissue. Artificial ventilation provides appropriate oxygenation; keeping the patient paralyzed decreases the body's metabolic needs.

Which types of damage can cause visceral pain? Select all that apply. Inflammatory chemicals Direct pressure Distention Strong contractions Ischemia

Distention Strong contractions Ischemia Explanation: Visceral pain is caused by strong contractions, distention, or ischemia affecting organ walls. There are fewer nociceptors in the viscera than in the skin. Viscera that are inflamed will respond to the distention of inflammation, not the chemicals that cause inflammation.

A client with Alzheimer disease (AD) is forgetful and has started to lose interest in social activities. Which treatment routine would be beneficial for the client? Memantine Donepezil High protein diet Vitamin D

Donepezil The cholinesterase inhibitor donepezil has been effective in slowing cognitive decline in early stages of AD. Memantine is used for moderate to severe AD. Vitamin D and a diet high in protein have not been shown to impact AD, although there is some promising research indicating that anti-inflammatory elements such as Vitamins E and C and ginkgo biloba may have neuroprotective properties.

A client with Parkinson disease presents with bradykinesia and an altered gait. These symptoms arise in response to the progressive deterioration of which structure in the brain? Dopamine nigrostriatal system Serotonergic system Cerebellum Limbic system

Dopamine nigrostriatal system

A client newly diagnosed with pancreatic cancer is admitted to begin treatment. Which pain descriptors can be associated with adenocarcinomas of the pancreas? Severe, intractable headache caused by excess fluid in the CNS causing cerebral edema. Dull epigastric pain accompanied by back pain, worse when lying flat and relieved by sitting forward. Sharp, stabbing pain with each deep breath. Abdominal pain following eating a large meal heavy with fat and protein.

Dull epigastric pain accompanied by back pain, worse when lying flat and relieved by sitting forward.

A child is being seen in the emergency department (ED) after ingesting crayons with lead in them. He is disoriented and having seizures. The provider suspects he has which of the following? Bacterial meningitis Viral meningitis Meningioma Encephalitis

Encephalitis Less frequent causes of encephalitis include ingesting toxic substances such as lead. People experience neurologic disturbances such as lethergy, disorientation, seizures, focal paralysis, delirium and coma.

A client has sustained damage to cranial nerve VIII. The nurse recognizes that the client may experience difficulty with: Taste Hearing Motor Smell

Hearing Explanation: Cranial nerve VIII (vestibulocochlear nerve) is associated with hearing. The other options involve different nerves.

A client has sustained damage to cranial nerve VIII. The nurse recognizes that the client may experience difficulty with: Hearing Motor Smell Taste

Hearing Explanation: Cranial nerve VIII (vestibulocochlear nerve) is associated with hearing. The other options involve different nerves.

A client tells the nurse that he is concerned about developing hepatitis after being exposed to contaminated feces, saliva, and food. The nurse is aware that the client is at risk for: Hepatitis D Hepatitis B Hepatitis C Hepatitis A

Hepatitis A

A health care provider suspects a client may have developed pancreatitis. Which laboratory value will confirm this diagnosis? Change in platelet count and prothrombin level High serum amylase and lipase Altered alkaline phosphatase and red blood cell count Chymotrypsin level and fibrinogen level

High serum amylase and lipase

A nurse is assessing a client with symptoms of botulism. The nurse will question the client regarding ingestion of which food? Dairy products Home-grown and canned vegetables Fresh fish Unrefrigerated meat

Home-grown and canned vegetables

A client is unable to stick out his tongue as a result of injury to cranial nerve XII. The nurse recognizes that the client has sustained as damage to which nerve? Hypoglossal Trigeminal Gustatory Vestibular

Hypoglossal Explanation: The name of cranial nerve XII is the hypoglossal nerve. It is responsible for the client's ability to stick out his or her tongue without deviation.

The nurse is caring for the client with hepatocellular carcinoma. Which of these does the nurse recognize is a cause of this disease? Exposure to non-steroidal anti-inflammatory medications Diabetes Gallbladder disease Illness with hepatitis B or C

Illness with hepatitis B or C

A 9-year-old girl has a diffuse collection of symptoms that are indicative of deficits in endocrine and autonomic nervous system control. She also suffers from persistent fluid and electrolyte imbalances. On which aspect of the nervous system listed below would her health care providers focus their diagnostic efforts? Potential damage to the girl's cerebellum Impaired function of her hypothalamus Possible damage to her pons and medulla Her afferent and efferent cranial nerve function

Impaired function of her hypothalamus Explanation: The hypothalamus plays a central role in the maintenance of fluid and electrolyte balance and in the maintenance of endocrine control. Various cranial nerves, the hindbrain, and the cerebellum would be less likely to be implicated.

When caring for the client with Laennec's cirrhosis, the nurse recognizes which pathophysiologic finding to be an expected etiology of jaundice? Decreased oxygen at birth Impaired uptake of billirubin Increased red cell development Increased conjugation of billirubin

Impaired uptake of billirubin

A resident of a long term care facility has contracted Clostridium difficile with frequent diarrhea and hyperactive bowel sounds. What process in the small intestine is most likely accompanying his current status? Pathogenic microorganisms are causing dilation of his small intestine, increasing motility. Segmentation waves have become more frequent as a result of his infection. Inflammation is accompanied by an increase in peristaltic movements of his small intestine. Intestinal stasis brought on by infection is preventing his small intestine from sufficiently slowing the rate of motility.

Inflammation is accompanied by an increase in peristaltic movements of his small intestine.

A 22 year-old student has developed a fever and diarrhea while on a backpacking trip in Southeast Asia. His oral temperature is 101.40 F. The diarrhea is bloody, frequent, and small in volume. These clinical manifestations are sufficiently distressing that he/she is visiting a local medical clinic in the area. Which of the following diagnoses best characterizes this health problem? Inflammatory diarrhea Factitious diarrhea Noninflammatory diarrhea Secretory diarrhea

Inflammatory diarrhea

Reflexes are basically "hard-wired" into the CNS. Anatomically, the basis of a reflex is an afferent neuron that synapses directly with an effector neuron to cause muscle movement. Sometimes the afferent neuron synapses with what intermediary between the afferent and effector neurons? Neurotransmitter Suprasegmental effectors Interneuron Intersegmental effectors

Interneuron

The emergency room doctor suspects a client may have bacterial meningitis. The most important diagnostic test to perform would be: Sputum culture Blood cultures Lumbar puncture CT of the head

Lumbar puncture The diagnosis of bacterial meningitis is confirmed with abnormal CSF findings. Lumbar puncture findings, which are necessary for accurate diagnosis, include a cloudy and purulent CSF under increased pressure.

A client reports a sudden intense headache. Which factor would indicate the presence of a possible subarachnoid hemorrhage? Sleep disturbance at night Family history of migraine Intractable pain History of smoking

Intractable pain Explanation: Subarachnoid hemorrhage causes a severe intractable headache. Headaches that disturb sleep or occur with exercise or sexual activity may be caused by neurologic lesions. Migraine headaches tend to run in families but do not contribute to hemorrhage.

Colonic microorganisms play a role in the synthesis of which of the following vitamins? C E D K

K

A client has been diagnosed with Guillain-Barré syndrome and is being treated in the intensive care unit. The client is experiencing rapidly ascending paralysis. What is the nurse's priority intervention when caring for this client? Inserting a Foley catheter Maintenance of the client's airway Restarting the IV when it infiltrates Assessing distal peripheral pulses

Maintenance of the client's airway

What role do the basal ganglia play in cognition? Monitor sensory information coming into the brain and apply it to information stored in the memory Make it possible for the motor neurons to innervate a few to thousands of muscle fibers and provide gross motor movement Initiate movement to avoid hazardous situations, while the stretch receptors serve to integrate motor movement so they function in a coordinated manner Provide central control over the gamma motor neurons to increase or decrease muscle tone in anticipation of changes in muscle force

Monitor sensory information coming into the brain and apply it to information stored in the memory

A client experiencing phantom limb pain after a traumatic amputation finds that opioids, biofeedback, a transcutaneous electrical nerve stimulation unit, and relaxation therapy are all needed to reduce the pain to a tolerable level. Which pain theory is best supported by this event? Gate control Neuromatrix Specificity Pattern

Neuromatrix Explanation: The neuromatrix theory joins multiple sources of input to aid in understanding the multiple dimensions of pain experience and behavior in chronic pain, along with other complex pain phenomena such as phantom limb pain. The gate control theory proposes that you can block pain by sending a different signal such as cold or pressure that will close a gate in the spinal cord to prevent transmission of the pain signal. The specificity theory states that pain is experienced when specific sensors are stimulated. The pattern theory is a collection of theories proposing that not only do pain receptors share pathways with other senses but that different activity patterns indicate whether pain is present or not.

Which intervention is usually the first line of therapy when treating moderate pain in the older adult population? Antihistamines like diphenhydramine Muscle relaxants like cyclobenzaprine Opioids, such as codeine Nonopioids, such as acetaminophen

Nonopioids, such as acetaminophen Explanation: Nonopioids are generally the first like of therapy for mild to moderate pain in the older adult population. Opioids are used for more severe pain and palliative care. Diphenhydramine is used primarily for allergy symptoms. Muscle relaxants like cyclobenzaprine are helpful for aching muscles.

A client diagnosed with Parkinson disease is displaying the following manifestations: tremor, rigidity, and slowness of movement. The nurse would interpret these as: Manifestations of another disease process Signs of clinical deterioration Normal manifestations of Parkinson disease Signs of clinical improvement

Normal manifestations of Parkinson disease

When assessing a client with acute cholecystitis, the nurse anticipates the client's report of pain will be consistent with which of these descriptions? Pain in the left lower quadrant, radiating to the back Pain that starts as a diffuse ache and localizes over 24 to 48 hours Pain in the right upper quadrant referred to the same shoulder Pain in the back, radiating to the groin

Pain in the right upper quadrant referred to the same shoulder

A patient is not able to absorb vitamin B12. The nurse determines that the patient is deficient in: Peptic (chief cells), which secrete pepsinogen S cells, which secrete secretin Mucous neck cells, which secrete mainly mucus Parietal (oxyntic) cells, which secrete HCl and intrinsic factor

Parietal (oxyntic) cells, which secrete HCl and intrinsic factor

What disease results from the degeneration of the dopamine nigrostriatal system of the basal ganglia? Guillain-Barré syndrome Huntington disease Myasthenia gravis Parkinson disease

Parkinson disease

A nurse assesses a client with a cerebral infarct for sensation. Which result indicates that second-order neurons are intact? Patellar reflex +2 Flaccid hemiparesis Alert and oriented x3 Dysphagia

Patellar reflex +2 Explanation: First-order neurons transmit sensory information from the periphery of the neurons to the central nervous system. Second-order neurons communicate directly with the thalamus and work with the reflex networks and sensory pathways in the spinal cord. Third-order neurons relay information from the thalamus to the cerebral cortex. Level of consciousness does not indicate sensation.

Guillain-Barré syndrome is characterized by which form of neuron damage? Transneuropathy Mononeuropathy Aneuropathy Polyneuropathy

Polyneuropathy

A adult has been diagnosed with a gastroesophageal reflux disease, in which the function of his lower esophageal sphincter is compromised. Which consequence of this condition is most likely? Decreased absorption of ingested foods and fluids. Impaired control of the gastric emptying rate. Protrusion of the stomach or regurgitation of stomach contents into the esophagus. Inappropriate release of gastric enzymes.

Protrusion of the stomach or regurgitation of stomach contents into the esophagus.

The parent of a toddler with Duchenne muscular dystrophy reports that the child has an increase in muscle size but a decrease in strength. The nurse documents this using which medical term? Pseudohypertrophy Chorea Fasciculations Dysdiadochokinesia

Pseudohypertrophy

A nurse caring for a client with multiple sclerosis notes that the client has mood swings. Which cause can best explain this? Depression over new diagnosis Psychological manifestation due to involvement of white matter of cerebral cortex A side effect of treatment Likely a preexisting mental illness

Psychological manifestation due to involvement of white matter of cerebral cortex

An anatomy student explains that the funnel-shaped portion of the stomach that connects with the intestine is called which region? Fundus Pyloric region Cardiac region Body

Pyloric region

Which complication of spinal cord injury is the most preventable in a paraplegic client? Skin breakdown Deep vein thrombosis Autonomic dysreflexia Muscle atrophy

Skin breakdown

A nurse explains to her patient that food is moved along the gastrointestinal (GI) tract with intermittent contractions that mix the food and move it along. These movements are found in which of the following organs? Small intestine Internal anal sphincter Upper region of the stomach Ileocecal valve

Small intestine

A patient is admitted to the hospital for further evaluation of problems experienced with gastrointestinal digestion and absorption. The nurse anticipates the health care provider will further assess the: Small intestine Large bowel Intestinal lumen Sigmoid colon

Small intestine

Through what specific component do neurotransmitters exert their action? Water Oxygen Specific proteins Carbon dioxide

Specific proteins Explanation: Neurotransmitters exert their actions through specific proteins, called receptors, embedded in the postsynaptic membrane. Water, carbon dioxide, and oxygen enter the brain with relative ease.

A client who is being treated for chronic low back pain is using a TENS unit for relief of pain. The nurse is aware that the use of this device is considered what type of pain relief? Biofeedback Distraction Stimulus-induced analgesia A physical agent

Stimulus-induced analgesia Explanation: Stimulus-induced analgesia is one of the oldest known methods of pain relief. Electrical stimulation methods of pain relief include TENS, electrical acupuncture, and neurostimulation. A physical agent would be the use of heat or cold. Distraction is a nonpharmacologic method of reduction of pain by distracting the client from his pain. Biofeedback is used to provide feedback to a person concerning the current status of some body function, temperature, temporal artery pulsation, blood pressure, or muscle tension

The production of chyme occurs in which of the following organs? Pancreas Small intestine Esophagus Stomach

Stomach

The nurse learns that different types of headaches respond to different therapies. Which headache is most responsive to nonpharmacologic therapy? Tension Cluster Migraine Sinus

Tension Explanation: Tension-type headaches often are more responsive to nonpharmacologic techniques, such as biofeedback, massage, acupuncture, relaxation, imagery, and physical therapy, than other types of headache. For people with poor posture, a combination of range-of-motion exercises, relaxation, and posture improvement may be helpful. The other options are usually best treated with medications that focus on the cause of the pain.

When explaining the role of liver Kupffer cells to a group of nursing students, which of the following statements about the function of these cells is most accurate? The cells are capable of removing and phagocytizing old and defective blood cells. These cells are the functional unit of the liver and are responsible for all liver secretions. The primary function of Kupffer cells is to secrete bile. The role of the Kupffer cells is to provide at least 50% of cardiac output each minute to each lobular of the liver.

The cells are capable of removing and phagocytizing old and defective blood cells.

A client with a systemic infection has been given strong doses of broad-spectrum antibiotics for a period of 10 days. The infection recedes and he is taken off the antibiotic therapy. Two days later he presents with a Clostridium difficile infection. How should the nurse best interpret this situation? C. difficile is part of the normal flora and rebounded after antibiotic therapy. The client's antibiotics disrupted the microbial balance and allowed overgrowth. The antibiotics were not specific enough, and C. difficile was allowed to survive. The current infection is unrelated and was coincidental to the time of the previous infection.

The client's antibiotics disrupted the microbial balance and allowed overgrowth.

A 60-year-old office worker has presented to a clinic reporting recent episodes of lower back pain. Which component of the client's assessment and history would be most indicative of a serious pathologic process? When supine, passive raising of the leg to 90° results in hamstring pain. The client's onset of pain has been gradual and the client has no prior history of back problems. The client has needed regular nonsteroidal anti-inflammatory drugs to control the pain in recent weeks. The client's pain is unrelieved by extended bed rest.

The client's onset of pain has been gradual and the client has no prior history of back problems.

Which of the following statements most accurately describes the function of the secretory glands in the gastrointestinal (GI) tract? Each day approximately 2000 mL of fluid is secreted into the GI tract. Secretory activity is increased with sympathetic stimulation. The secretions are mainly albumin with sodium and potassium. The secretory glands produce mucus to lubricate and protect the mucosal layer of the GI tract wall.

The secretory glands produce mucus to lubricate and protect the mucosal layer of the GI tract wall.

A mother is placing her child into the bathtub. The child immediately jumps out of the tub and begins to cry, stating her feet are "burning." The nurse in the emergency department knows that the child's response is based on which pathophysiologic principle listed below? Child react much quicker to contact with hot water than adults. The thermal processing center is located on the rapid-conducting anterolateral system on the same side of the brain as the injury. The tactile sensation occurs well in advance of the burning sensation. The local withdrawal reflex reacts first. It takes a long time for thermal signals to be processed before the brain can send a signal through the spinal cord and tell the foot to withdraw.

The tactile sensation occurs well in advance of the burning sensation. The local withdrawal reflex reacts first. Explanation: If a person places a foot in a tub of hot water, the tactile sensation occurs well in advance of the burning sensation. The foot has been removed from the hot water by the local withdrawal reflex well before the excessive heat is perceived by the forebrain.

A client who is being seen in the outpatient clinic reports a single episode of unilateral arm and leg weakness and blurred vision that lasted approximately 45 minutes. The client is most likely experiencing: Cardiogenic embolic stroke Transient ischemic attack (TIA) Lacunar infarct Thrombotic stroke

Transient ischemic attack (TIA) Transient ischemic attacks are brief episodes of neurologic function resulting in focal cerebral ischemia not associated with infarction that usually resolve in 24 hours. The causes of transient ischemic attack are the same as they are for stroke. Embolic stroke usually has a sudden onset with immediate maximum deficit. Lacunar infarcts produce classic recognizable "lacunar syndromes" such as pure motor hemiplegia, pure sensory hemiplegia, and dysarthria with clumsy hand syndrome.

A 21-year-old male is brought to the ED following a night of partying in his fraternity. His friends found him "asleep" and couldn't get him to respond. They cannot recall how many alcoholic beverages he drank the night before. While educating a student nurse and the man's friends, the nurse begins by explaining that alcohol is: a water-soluble compound that is easily absorbed by the gastric lining of the stomach. Very lipid-soluble and rapidly crosses the blood-brain barrier. very likely to cause sedation and therefore the client just needs to sleep it off. able to reverse the transport of some substances to remove them from the brain.

Very lipid soluble and rapidly crosses the blood-brain barrier. Explanation: The blood-brain barrier prevents many drugs from entering the brain. Most highly water-soluble compounds are excluded from the brain. Many lipid-soluble molecules cross the lipid layers of the blood-brain barrier with ease. Alcohol, nicotine, and heroin are very lipid-soluble and rapidly enter the brain. Alcohol toxicity can kill clients, especially if they are not used to consuming beverages. These clients should never be left alone to "sleep it off."

A client with a history of migraine headaches tells the physician that he or she usually experiences an aura before the onset of the headache. The client is most likely experiencing: Lethargy Visual disturbances Dysphasia Dizziness

Visual disturbances Migraine aura is associated with visual symptoms, including flickering lights, spots, or loss of vision; sensory symptoms, including feeling of pins or needles, or numbness; and speech disturbances or other neurologic symptoms.

Which type of reflex is stimulated by a nociceptive stimulus? Carotid sinus baroreflex Stretch reflex Withdrawal reflex Myotatic reflex

Withdrawal reflex Explanation: The withdrawal reflex is stimulated by a damaging (nociceptive) stimulus. The myotatic, or stretch reflex, controls muscle tone and helps maintain posture. The carotid sinus baroreflex assists adjustment of the cardiovascular system at the level of the brain stem.

A rare condition caused by gastrin-secreting tumors most commonly found in the small intestine or pancreas is called which of the following? Stevens-Johnson syndrome Sickle cell anemia Zollinger-Ellison syndrome Creutzfeldt-Jakob disease

Zollinger-Ellison syndrome

The family of an older adult reports increasing inability to perform basic activities of daily living. After evaluation, the client is diagnosed with Alzheimer's disease. What intervention will be implemented to slow cognitive decline? Psychotherapy Antioxidant therapy Lipid-lowering agents Cholinesterase inhibitors

cholinesterase inhibitors Cognitive function in Alzheimer's disease (AD) can be enhanced by the use of medications. The acetylcholinsterase inhibitors donepezil, rivastigmine, and galantamine all increase concentration of acetylcholine in the cerebral cortex. -There has been no demonstrated improvement of cognitive function with use of lipid-lowering statins or antioxidant nutritional supplement therapy. -Psychotherapy is appropriate for depression.

The parents of an infant born with hydrocephalus are concerned about the size of the baby's head. The doctors are telling them that the infant needs the surgical placement of a shunt. The nurse caring for the infant in the neonatal intensive care unit explains that placement of a shunt will: not affect the size of the infant's head. decrease the likelihood of further neurological deficits. increase intracranial pressure. reverse any neurologic deficits that are present.

decrease the likelihood of further neurological deficits. The placement of a shunt to drain excess cerebrospinal fluid decreases intracranial pressure and thereby decreases the likelihood of further neurologic deficits. The placement of a shunt may or may not affect the size of the infant's head or reverse neurologic deficits that may have already occurred.

The common bile duct opens into which of the following parts of the gastrointestinal tract? Duodenum Ileum Cecum Jejunum

duodenum

A client has developed global ischemia of the brain. The nurse determines this is: inadequate perfusion of the right side of the brain. inadequate perfusion to the dominant side of the brain. inadequate perfusion of the nondominant side of the brain. inadequate to meet the metabolic needs of the entire brain.

inadequate to meet the metabolic needs of the entire brain Global ischemia occurs when blood flow is inadequate to meet the metabolic needs of the entire brain. The result is a spectrum of neurologic disorders reflecting diffuse brain dysfunction.

Feelings of dread, high anxiety, or exquisite pleasure can be elicited by stimulation of areas in which structure? Limbic system Cerebellum Occipital lobe Temporal lobe

limbic system Explanation: Stimulation of specific areas of the limbic system can lead to feelings of dread, high anxiety, or exquisite pleasure. The temporal lobe is involved with auditory functions, the cerebellum with proprioception, and the occipital lobe with visual function.

During a lecture about the function of the intestine related to food digestion, the faculty mentions that when the students consume foods high in acid, the intestines will: stimulate the release of secretin which then inhibits release of gastrin. stimulate the release of glucagon-like peptide 1 to lower blood glucose levels. inhibit the release of glucose-dependent insulinotropic peptide to slow gastric emptying. stimulate pancreatic enzymes to release more insulin.

stimulate the release of secretin which then inhibits release of gastrin.

Which individual is likely to have the best prognosis for recovery from his or her insult to the peripheral nervous system? An adult: who suffered a bone-depth laceration to the shoulder during a knife attack. client who had nerves transected during surgery to remove a tumor from the mandible. client who developed rhabdomyolysis and ischemic injury after a tourniquet application. who had his forearm partially crushed by gears during an industrial accident.

who had his forearm partially crushed by gears during an industrial accident.

The gastrointestinal laboratory nurse is learning about small intestine secretions. Which of the following explanations is most accurate? "The Brunner glands secrete large amounts of acidic mucus." "An extensive array of mucus-producing glands, called Brunner glands, is concentrated where contents from the stomach and secretions from the liver and pancreas enter the duodenum." "Sympathetic stimulation causes a marked increase in mucus production." "Ulcers are more likely to occur in the stomach than in the duodenum."

"An extensive array of mucus-producing glands, called Brunner glands, is concentrated where contents from the stomach and secretions from the liver and pancreas enter the duodenum."

The parent of an infant who developed hydrocephalus while in utero is very concerned that the child will have significant intellectual dysfunction. The best response to the parent would be: "Unfortunately, there usually is significant brain dysfunction." "Because the skull sutures are not fused there may be no brain damage." "The cranial sutures are fused and decrease brain damage." "Infants never have symptoms from hydrocephalus."

"Because the skull sutures are not fused there may be no brain damage." When hydrocephalus develops in utero, before the cranial sutures have fused, the head can swell and decrease intracranial pressure, thereby decreasing the amount of brain tissue that is compressed

A client is having difficulty with sleeping and has also been experiencing marital difficulties over the past couple of months. The client tells the nurse at the physician's office that all this started after he had a car accident earlier that year. What would be the most important question for the nurse to ask? "Did you sustain any injuries in the accident?" "What is your normal routine before bedtime?" "How long have you been married?" "Did you go to the hospital following the accident?"

"Did you sustain any injuries in the accident?" Postconcussion syndrome can interfere with daily living and also with relationships and can continue for months. The syndrome can include amnesia, insomnia, headache, difficulty concentrating, and irritability. In this situation, it would be very important to determine if the patient sustained a head injury to rule out postconcussion syndrome.

The nurse is working with a client who has been diagnosed with recurring migraine headaches. Which advice by the nurse is most appropriate? "Migraine headaches usually respond poorly to medication, so I can help you explore nonpharmacologic options." "Migraines are poorly understood and are generally considered unavoidable, so always have your medication with you." "Many people find that maintaining regular eating and sleeping habits is beneficial." "It's likely in your best interest to move to a rural setting and explore getting a less stressful job."

"Many people find that maintaining regular eating and sleeping habits is beneficial." Explanation: Nonpharmacologic treatment includes the avoidance of migraine triggers, such as foods, that precipitate an attack. Many persons with migraines benefit from maintaining regular eating and sleeping habits. The client may need to change her lifestyle, but not to the extent of moving and changing jobs. Migraines are avoidable, most of the time, if individuals adhere to their diets and watch the triggers. Medication is very useful for most clients.

A gastroenterologist is teaching a group of students about the enteric nervous system in preparation for a consult with a client who has experienced a spinal cord injury. Which of the physician's teaching points is most accurate? "Sympathetic innervation of much of the GI tract occurs by way of the vagus nerve." "Parasympathetic stimulation blocks the release of the excitatory neuromediators and inhibits GI motility." "The enteric nervous system is made up of the myenteric and submucosal plexuses; these are located in the wall of the GI tract." "The myenteric plexus is responsible for controlling the function of each segment of the intestinal tract."

"The enteric nervous system is made up of the myenteric and submucosal plexuses; these are located in the wall of the GI tract."

A male patient has just been diagnosed with esophageal cancer. He knew he was losing weight and fatigued most days, but he just attributed it to aging and working. The physician recommends chemotherapy and irradiation. However, the cancer has already metastasized. The patient asks the nurse what he can expect if he agrees to the treatments. The nurse responds: "These therapies will most likely cure your cancer." "You need to talk with your physician some more. I will page him for you." "The therapies may shrink the cancer in an effort to increase survival of the cancer." "The doctor is prescribing treatment measures to help you swallow better."

"The therapies may shrink the cancer in an effort to increase survival of the cancer."

A patient takes 650 mg of aspirin every 4 hours daily for complaints of joint pain. Which of the following statements should be included in the patient's teaching plan? "This medication will not assist with joint pain." "You should have your partial thromboplastin time assessed." "You should have your hemoglobin and hematocrit checked weekly." "This medication can damage gastric mucosa."

"This medication can damage gastric mucosa."

A 51 year-old male professional is in the habit of consuming 6 to 8 rum and cokes each evening after work. He assures the nurse practitioner who is performing his regular physical exam that his drinking is under control and does not have negative implications for his work or family life. How could the nurse best respond to the client's statement? "In spite of that, the amount of alcohol you are drinking is likely to result first in cirrhosis and, if you continue, in hepatitis or fatty liver changes." "That may be the case, but you are still creating a high risk of hepatitis A or B or liver cancer." "When your body has to regularly break down that much alcohol, your blood and the functional cells in your liver accumulate a lot of potentially damaging toxic byproducts." "You are more than likely inflicting damage on your liver, but this damage would cease as soon as you quit drinking."

"When your body has to regularly break down that much alcohol, your blood and the functional cells in your liver accumulate a lot of potentially damaging toxic byproducts."

A 71 year-old male has been recently diagnosed with a stage III tumor of colorectal cancer, and is attempting to increase his knowledge base of his diagnosis. Which of the following statements about colorectal cancer demonstrates a sound understanding of the disease? "If accurate screening test for this type of cancer existed, it could likely have been caught earlier." "A large majority of patients who have my type of colon cancer survive to live many more years." "The NSAIDs and aspirin that I've been taking for many years probably contributing to my getting cancer." "While diet is thought to play a role in the development of colorectal cancer, the ultimate causes are largely unknown."

"While diet is thought to play a role in the development of colorectal cancer, the ultimate causes are largely unknown."

The nurse suspects a spinal cord injury client is developing autonomic dysreflexia. Which assessment findings would confirm the development of this complication? Select all that apply. Complains of a pounding headache Cold, cyanotic lower legs. BP 180/98 Pulse rate 49 Skin covered with macular rash

-Complains of a pounding headache. -BP 180/98. -Pulse rate 49. Explanation: Autonomic dysreflexia represents an acute episode of exaggerated sympathetic reflex responses that occur in people with injuries at T6 and above, in which CNS control of spinal reflexes is lost. It is characterized by hypertension (BP 180.98), skin pallor, vagal slowing of the heart rate (pulse 49), and h

A client has just begun to experience an ischemic stroke. The blood supply from the middle cerebral artery is being blocked by a large blood clot. How long before brain cells begin to die due to lack of ATP? 20 minutes 30 to 40 minutes 4 to 6 minutes 10 seconds

4 to 6 minutes Explanation: Without oxygen, brain cells continue to function for approximately 10 seconds, and the death of brain cells begins within 4 to 6 minutes. Twenty minutes is the time of cardiac cell death, and 3 hours is the window of time that thrombolytic medications can be safely used. Testing is done at each segmental level, or dermatome, moving upward along the body and neck from coccygeal segments through the high cervical levels to test the functional integrity of all the spinal nerves.

Which principle best explains symptoms of amyotrophic lateral sclerosis (ALS), including dysphagia, muscle weakness and spasticity, and dysphonia? ALS is caused by disruption to the cerebellum. ALS is caused by muscular necrosis. ALS is caused by lack of dopamine in the body. ALS is caused by both an upper motor neuron and lower motor neuron disturbance.

ALS is caused by both an upper motor neuron and lower motor neuron disturbance.

Absorption is a major function of the GI tract. How is absorption accomplished in the GI tract? Active transport and osmosis Active transport and diffusion Osmosis and diffusion Diffusion and inactive transport

Active transport and diffusion

A 79-year-old woman reports a recent onset of "nearly constant heartburn." During the assessment interview, she states that she has "lots of aches and pains." She states that she is not on any prescription medications but often takes aspirin for pain. The nurse should suspect what diagnosis? Acute gastritis Gastric cancer Helicobacter pylori infection Staphylococcal infection

Acute gastritis

A client has just been diagnosed with multiple sclerosis (MS). The nurse recognizes that the client's condition is a result of: Circulating antibodies attacking the postsynaptic acetylcholine levels Degenerative changes in the musculoskeletal system Degeneration of the lower cell bodies of the lower motor neurons in the gray matter An immune-mediated response that is caused by the demyelinization of the myelin sheath of the white matter of the brain, spinal cord, and optic nerve

An immune-mediated response that is caused by the demyelinization of the myelin sheath of the white matter of the brain, spinal cord, and optic nerve

The mother of a 19 week-old infant has brought her baby in for assessment to a pediatrician because of the baby's persistent weight loss and diarrhea. An intestinal biopsy has confirmed a diagnosis of celiac disease, and the child's mother is anxious to know what caused the disease. Which of the following aspects of the etiology of celiac disease would underlie the explanation that the physician provides? Bacterial or chemical invasion of the peritoneum leads to decreased nutrient absorption and transport. An inappropriate T-cell mediated response results in increased levels of antibodies and an inflammatory response. Neurogenic or muscular inhibition of peristalsis results in inappropriate motility of ingested food in the lower small intestine and the colon. Inability to process or absorb the fat content of breast milk results in malnutrition and deficiency of fat-soluble vitamins.

An inappropriate T-cell mediated response results in increased levels of antibodies and an inflammatory response.

What medication teaching should be done for a woman of childbearing age with a seizure disorder? Some antiseizure drugs can interfere with vitamin K metabolism. Antiseizure drugs do not interact with oral contraceptives. Antiseizure drugs increase the risk for congenital abnormalities. All women of childbearing age should be advised to take a vitamin C supplement.

Antiseizure drugs increase the risk for congenital abnormalities. For women with epilepsy who become pregnant, antiseizure drugs increase the risk for congenital abnormalities and other perinatal complications. - Many of the antiseizure medications interact with oral contraceptives and can interfere with vitamin D metabolism. -All woman should be advised to take folic acid supplementation.

The nurse is caring for a client with a brain tumor when the client begins to vomit. Which intervention should the nurse do first? Assess for signs/symptoms of cerebrovascular accident (stroke). Contact physician for anti-nausea medication orders. Document the finding as it is an expected symptom. Assess for other signs/symptoms of increased intracranial pressure.

Assess for other signs/symptoms of increased intracranial pressure The tumor may be causing increased intracranial pressure. Vomiting, with or without nausea, is a common symptom of increased intracranial pressure and/or brain stem compression. The nurse's first action is to assess for other signs/symptoms of increased intracranial pressure. Once the assessment is completed, the nurse should contact the physician if indicated by the findings.

A 47-year-old woman was diagnosed with amyotrophic lateral sclerosis 3 years ago and has experienced a progressive onset and severity of complications. She has been admitted to a palliative care unit due to her poor prognosis. What assessments and interventions should the nursing staff of the unit prioritize in their care? Assessment and documentation of cognitive changes, including confusion and restlessness Regular pain assessment and administration of opioid analgesics as needed Cardiac monitoring and administration of inotropic medications. Assessment of swallowing ability and respiratory status.

Assessment of swallowing ability and respiratory status.

The parasympathetic nervous system is part of which system? Thoracolumbar nervous system Central nervous system Autonomic nervous system Somatic nervous system

Autonomic nervous system Explanation: The efferent outflow from the autonomic nervous system has two divisons: the sympathetic nervous system and the parasympathetic nervous system.

A practitioner is preparing to do a lumbar puncture on a client with suspected meningitis. Which area on the spine does the practitioner choose to obtain a cerebrospinal fluid (CSF) sample? Between L3 and L4 Between C7 and C8 Between T1 and T2 Between T12 and L1

Between L3 and L4 Explanation: The adult cord usually terminates at the inferior border of L1. The arachnoid and its enclosed subarachnoid space, which is filled with CSF, do not close down on the filum terminale until they reach the second sacral vertebra. This results in a formation of a pocket of CSF, the dural cisterna spinalis, which extends from approximately L2 to S2. Because this has an abundant supply of CSF and the spinal cord does not extend this far, the area (L3 or L4) is often used to perform a lumbar puncture to obtain CSF.

Following a collision while mountain biking, the diagnostic work up of a 22-year-old male has indicated the presence of an acute subdural hematoma. Which pathophysiologic process most likely underlies his diagnosis? A traumatic lesion in the frontal or temporal lobe has resulted in increased ICP. Vessels have burst between the client's skull and his dura. Blood has accumulated between the man's dura and subarachnoid space. Blood has displaced CSF in the ventricles as a consequence of his coup-contrecoup injury.

Blood has accumulated between the man's dura and subarachnoid space. A subdural hematoma develops in the area between the dura and the arachnoid space while epidural hematomas exist between the skull and dura.

A soccer player has been diagnosed with a brain contusion after being injured in a game. The best explanation of the injury by the nurse would be that: tearing of brain tissue occurred. bruising on the surface of the brain occurred. hypoxia to the brain occurred. transient neurogenic dysfunction caused by mechanical force to the brain occurred.

Bruising on the surface of the brain occurred. Contusions represent bruising on the surface of the brain, and lacerations are a tearing of brain tissue.

The nurse on the pediatric unit is implementing distraction strategies for a child who is experiencing pain. Which strategies would be best for the nurse to implement? Select all that apply. Games Acetaminophen Music Bubbles Television Silence

Bubbles Music Television Games Explanation: Distraction helps children of any age divert their attention away from pain and onto other activities. Common diversions include bubbles, music, television, conversation, and games.

A client diagnosed with inflammatory diarrhea is having multiple small, bloody stools with a fever. Which could be a likely cause of this inflammatory diarrhea? S. aureus H. pylori M. tuberculosis C. difficile

C. difficile

A client is brought into the emergency room after falling from a roof and landing on his chin and face. With this type of spinal extension injury, the nurse anticipates which location of the spinal cord will most likely be injured? L2-L4 C4-C6 C2-C3 T4-T8

C4-C6

At what level of the cervical spine would a complete cord injury prevent flexion and extension of the fingers? C7 C6 C5 C8

C8

A nurse educator is explaining basic neuroanatomy to a class of prospective nursing students. Which statement best conveys an aspect of the role of cerebrospinal fluid (CSF)? CSF cushions the brain and provides a near-water medium for diffusion of nutrients. CSF distributes plasma proteins throughout the superficial gray matter of the CNS. It provides physical protection for the brain and ensures leukocytes and erythrocytes are evenly distributed in the CNS. It ensures the high metabolic and oxygenation needs of the brain are met, as well as absorbing physical shocks.

CSF cushions the brain and provides a near-water medium for diffusion of nutrients. Explanation: In addition to providing a cushion for the CNS, CSF provides a medium that is 99% water in which nutrients, electrolytes, and wastes can be diffused. It is not centrally involved in the distribution of oxygen, plasma proteins, or blood cells.

A client reports "the worst headache" of her life with associated blurred vision. Subsequent diagnostic testing has resulted in a diagnosis of an intra-axial brain tumor. What other clinical manifestations would be consistent with this diagnosis? Select all that apply. Seizure activity Tinnitus and earaches, with no accompanying signs of infection Nausea and vomiting Recurrent epistaxis (nosebleeds) Cognitive and personality changes

Cognitive and personality changes Seizure activity Nausea and vomiting The clinical manifestations of brain tumors depend on the size and location of the tumor. General signs and symptoms include headache, nausea, vomiting, mental changes, papilledema, visual disturbances (e.g., diplopia), alterations in sensory and motor function, and seizures. Tinnitus, ear pain, and epistaxis are not among the more common signs and symptoms of a brain tumor.

What will the nurse teach a client with trigeminal neuralgia about the condition? It is good to spend time alone every day. Surgery will relieve the condition. Carbamazepine is a first-line treatment. Avoid eating if it triggers the pain.

Carbamazepine is a first-line treatment. Explanation: Trigeminal neuralgia is a condition in which clients experience brief, severe, repetitive lightninglike or throbbing pain along the distribution of one or more of the branches of the fifth cranial nerve. Clients should be taught to avoid triggers when possible, but although eating may be a trigger, it is necessary for the client to maintain nutrition. Social isolation is a possible problem arising from the condition. Time alone does not contribute significantly to improvement of the condition. Although treatment rarely provides total relief, trigeminal neuralgia can be controlled with carbamazepine and surgical release of vessels, nerve roots, or scar tissue. If other treatments are ineffective, partial destruction of the nerve branches with heat, balloon compression, or glycerol injection may be performed. Newer therapies to be considered are botulinum toxin injection and gamma radiation with stereotactic surgery.

Peripheral nerve disorders are not uncommon. What is an example of a fairly common mononeuropathy? Guillain-Barré syndrome Phalen maneuver Carpal tunnel syndrome Myasthenia gravis

Carpal tunnel syndrome

Crohn's disease is recognized by sharply demarcated, granulomatous lesions that are surrounded by normal-appearing mucosal tissue. The nurse recognizes these lesions to be defined by which of the following descriptions? Mosaic Pyradimal Triangular Cobblestone

Cobblestone

Crohn's disease is recognized by sharply demarcated, granulomatous lesions that are surrounded by normal-appearing mucosal tissue. The nurse recognizes these lesions to be defined by which of the following descriptions? Pyradimal Triangular Mosaic Cobblestone

Cobblestone

The nurse assessing a client with a traumatic brain injury assesses for changes in which neurologic component? Select all that apply. Sensory function Metabolic function Level of consciousness Motor function Cognition

Cognition Level of consciousness Motor function Sensory function

Select the function of the occipital lobe. Discrimination of sounds entering opposite ears Anticipation and prediction of consequences of behavior Determination of objects through the sense of touch Color, motion, and depth perception

Color, motion, and depth perception Explanation: The occipital lobe is associated with the ability to experience color, depth, and motion perception. The temporal lobe is responsible for behavior, and the parietal lobe helps us to determine objects through the sense of feel.

A health care provider suspects a client has developed diverticular disease. Which diagnostic test is usually prescribed to confirm the diagnosis? Positron-emission tomography (PET) scan Computed tomography (CT) scan Barium enema Flat-plate of the abdomen

Computed tomography (CT) scan

The cerebellum, separated from the cerebral hemispheres by the tentorium cerebelli, lies in the posterior fossa of the cranium. What is one of the functions of the cerebellum? Contains the main motor pathways between the forebrain and the pons Contains the pontine nuclei Coordinates smooth and accurate movements of the body Conveys the senses of pain, temperature, touch, and proprioception to the superficial and deep regions of the face

Coordinates smooth and accurate movements of the body Explanation: The cerebellum compares what is actually happening with what is intended to happen. It then transmits the appropriate corrective signals back to the motor system, instructing it to increase or decrease the activity of the participating muscle groups so that smooth and accurate movements can be performed.

A nurse is concerned about a patient's continual use of nonsteroidal anti-inflammatory drugs (NSAIDs). Which of the following is the nurse concerned about? Increase in the proton pump Increase in gastrin secretion Damage to the gastric mucosa Decrease in the production of intrinsic factor

Damage to the gastric mucosa

Which of the meninges provides the major protection for the brain and spinal cord? Arachnoid membrane Pia mater Dura mater Tentorium cerebelli

Dura mater Explanation: All surfaces of the spinal cord, brain, and segmental nerves are covered with a delicate connective tissue layer called the pia mater. A second, very delicate, nonvascular, and waterproof layer, called the arachnoid, encloses the entire central nervous system. Immediately outside the arachnoid is a continuous sheath of strong connective tissue, the dura mater, which provides the major protection for the brain and spinal cord. The tentorium cerebelli is the inner layer of the dura that anchors the brain to the skull.

A client who sustained a complete C6 spinal cord injury 6 months ago has been admitted to the hospital for pneumonia. The nurse observes the client with diaphoresis above the level of C6 and the blood pressure is 260/140 mm Hg. What is the first intervention the nurse should provide? Give the client some orange juice and sugar. Elevate the head of the bed. Insert an indwelling catheter. Disimpact the stool from the anal vault.

Elevate the head of the bed.

A teenager, exposed to West Nile virus a few weeks ago while camping with friends, is admitted with headache, fever, and nuchal rigidity. The teenager is also displaying some lethargy and disorientation. The nurse knows which medical diagnosis listed below may be associated with these clinical manifestations? Encephalitis Spinal infection Lyme disease Rocky mountain spotted fever

Encephalitis. Encephalitis represents a generalized infection of the parenchyma of the brain or spinal cord. A virus, such as West Nile virus, usually causes encephalitis, although it may be caused by bacteria, fungi, and other organisms. Encephalitis is characterized by fever, headache, and nuchal rigidity. However, more often, people also experience neurologic disturbances, such as lethargy, disorientation, seizures, focal paralysis, delirium, and coma. Rocky mountain spotted fever is a tickborne disease caused by the bacterium Rickettsia rickettsii and usually begins with a sudden onset of fever and headache. A rash may occur 2 to 5 days after fever onset. Lyme disease is also a tickborne disease. Typical symptoms include fever, headache, fatigue, and a characteristic skin rash called erythema migrans. As the disease progresses, the client develops bouts of severe joint pain and swelling of the joint. Neurologic problems may occur weeks, months, or even years after the infection and may include inflammation of the membranes surrounding your brain (meningitis). Spinal infections can be thought of as a spectrum of disease comprising spondylitis, discitis, spondylodiscitis, pyogenic facet arthropathy, and meningitis.

The nursing student is caring for a client with a pheochromocytoma and is aware that the release of catecholamines induces the hypertensive state. Which catecholamines are responsible for this high blood pressure? Select all that apply. Acetylcholine Epinephrine Dopamine Serotonin Norepinephrine

Epinephrine Norepinephrine Dopamine Explanation: The catecholamines, which include norepinephrine, epinephrine, and dopamine, are synthesized in the axoplasm of sympathetic nerve terminal endings from the amino acid tyrosine. Epinephrine accounts for approximately 80% of the catecholamine released from the adrenal gland. Acetylcholine is excitatory to most muscarinic receptors. Serotonin is an important chemical neurotransmitter in the human body. It is commonly regarded as a chemical that is responsible for maintaining mood balance.

Pharmacologic treatment for peptic ulcers has changed over the past several decades. The nurse knows that the goal for pharmacologic treatment is focused on which of the following? Eradicating Helicobacter pylori (H. pylori) Neutralizing blood count Promoting special diet Increasing acid production

Eradicating Helicobacter pylori (H. pylori)

Parents of a 20-month-old infant report that he refuses food or eats poorly and that he grimaces when he swallows. He also is irritable and cries a lot. The mother is worried that he ate something inappropriate this morning, because he vomited something that looked like coffee grounds. Which of the following health problems would the care team first suspect? Rotavirus infection Esophagitis from gastrointestinal reflux Appendicitis Hirschsprung's disease

Esophagitis from gastrointestinal reflux

Excessive activity of the excitatory neurotransmitters and their receptor-mediated effects is the cause of which type of brain injury? Ischemic Increased intercranial volume and pressure Hypoxic Excitotoxic

Excitotoxic Excitotoxicity is a final common pathway for neuronal cell injury and death. It is associated with excessive activity of excitatory amino acid neurotransmitters.

The underlying causative problem in Parkinsonism is: Viral infection Failure of dopamine release Genetic defect Autoimmune disorder

Failure of dopamine release

A client who is diagnosed with seizures describes feeling confused after experiencing a seizure. The family members report that the client has been smacking his lips prior to having a seizure. The client most likely experienced which type of seizure? Focal Clonic Atonic Myoclonic

Focal Focal seizures with impairment of consciousness sometimes referred to as psychomotor seizures are often accompanied by automatisms or repetitive nonpurposeful activities such as lip smacking, grimacing, patting, or rubbing clothing. Confusion during the postictal period (after a seizure) is common. The other seizures do not display these manifestations.

Helicobacter pylori gastritis is thought to be caused by a previous infection when the client was younger. Chronic gastritis caused by H. pylori can lead to which possible condition? Gastric atrophy Decreased risk of multiple myleoma Duodenal ulcer formation Decreased risk of gastric adenocarcinoma

Gastric atrophy

A patient is experiencing reflux of stomach contents into the esophagus. The nurse determines that the problem may result from: Pharyngoesophageal sphincter Pyloric sphincter Gastroesophageal sphincter Upper esophageal sphincter

Gastroesophageal sphincter

The chart of a client admitted because of seizures notes that the seizure activity began simultaneously in both cerebral hemispheres. The nurse should interpret this to mean that the client experienced: generalized seizure. unknown type of seizure. focal seizure with impairment of consciousness. focal seizure without impairment of consciousness.

Generalized seizure When seizure activity begins simultaneously in both cerebral hemispheres, it is considered a generalized seizure.

A client is having an upper endoscopy to determine the presence of a gastric ulcer. After the procedure is performed, the nurse instructs the client that he cannot have anything to eat or drink until the return of the gag reflex. Which nerve is the nurse testing for return of function? Hypoglossal Trigeminal Glossopharyngeal Abducens

Glossopharyngeal Explanation: The glossopharyngeal nerve innervates the stylopharyngeus muscle, posterior external ear, taste buds of posterior half of tongue, oral pharynx, parotid gland, and pharyngeal muscles. It affects the function of proprioception, somesthesia, taste, gag reflex, salivary reflex, and assists in swallowing.

The nurse measures a blood glucose level of 40 mg/dL (2.22 mmol/L) for a client with type 1 diabetes. Why would it be important for the nurse to institute an intervention to elevate the glucose level in this client? Small amounts of glucose may be stored in the brain for a short period of time but are rapidly metabolized. The nurse should administer a food source of protein in order to increase the glucose level for improved cardiac function. Glucose is not stored in the brain and is a major fuel source for brain function. It is not necessary for glucose to be replaced immediately because the client will have enough stored to function for a while.

Glucose is not stored in the brain and is a major fuel source for brain function. Explanation: Glucose is the major fuel source for the nervous system but neurons have no provision for storing glucose. Ketones can provide for limited temporary energy requirements. However, these sources are rapidly depleted.

In the balance of secretions in the gastric mucosa by the parietal cells, which of the following ions is produced to buffer the production of hydrochloric acid? H2O K+ HCO3- OH-

HCO3-

Maintenance of blood gas concentrations, water balance, and food consumption are controlled by which part of the brain? Basal ganglia Cerebellum Cerebral hemispheres Hypothalamus

Hypothalamus Explanation: The hypothalamus is the area of master level integration of homeostatic control of the body's internal environment. Maintenance of blood gas concentrations, water balance, food consumption, and major aspects of endocrine and autonomic nervous system control require hypothalamic function.

The nurse is planning an inservice on hypoxia versus ischemia in brain-injured clients. The nurse should include which of the following? Hypoxia produces a generalized depressive effect on the brain. Ischemia does not interfere with delivery of glucose. Ischemia denotes a deprivation of oxygen with maintained perfusion. Hypoxia denotes an interruption in blood flow.

Hypoxia produces a generalized depressive effect on the brain. Hypoxia denotes a deprivation of oxygen with maintained blood flow (perfusion), whereas ischemia is a situation of greatly reduced or interrupted blood flow. Hypoxia produces a generalized depressant effect on the brain. Ischemia interferes with delivery of oxygen and glucose as well as the removal of metabolic wastes.

A nurse working in a busy orthopedic clinic is asked to perform the Tinel sign on a client having problems in her hand/wrist. In order to test Tinel sign, the nurse should give the client which direction? Hold your wrist in complete flexion, keep it in this position for 60 seconds. How does your hand feel after placing it in a neutral position? Stand tall, arms at your side, shut your eyes; place the tip of your index finger to your nose. I'm going to tap this tuning fork; place it on the side of your thumb, then tell me what you are feeling in your hand and wrist. I'm going to tap (percuss) over the median nerve in your wrist; tell me what sensation you feel while I am doing this. Does the sensation stay in the wrist or go anywhere else?

I'm going to tap (percuss) over the median nerve in your wrist, tell me what sensation you feel while I am doing this. Does the sensation stay in the wrist or go anywhere else?

A nurse practitioner is providing care for a male client with a longstanding hiatal hernia. Which of the following statements most accurately captures an aspect of the pathophysiology of hiatal hernias? Paraoesophageal hiatal hernias are common and are normally not treated if the client is asymptomatic. An incompetent pyloric sphincter and high fat diet are commonly implicated in the development of hiatal hernias. The root causes of hiatal hernias are normally treatable with medication. If esophageal acid clearance is impaired, esophagitis can result.

If esophageal acid clearance is impaired, esophagitis can result.

The nurse is reviewing laboratory results of a client who has liver failure. Which finding would place the client at increased risk for bleeding? Increased prothrombin time Decreased number of red blood cells Increased levels of vitamin K Increased platelet count

Increased prothrombin time

When caring for the client with hepatic failure, the nurse recognizes which of these problems places the client at increased risk for bleeding? Decreased red blood cells Increased vitamin K Increased prothrombin time Increased platelet count

Increased prothrombin time

The geriatrician providing care for a 74-year-old man with diagnosis of Parkinson disease has recently changed the client's medication regimen. What is the most likely focus of the pharmacologic treatment of the man's health problem? Preventing axonal degradation of motor neurons M Maximizing acetylcholine release from synaptic vesicles at neuromuscular junctions '' Increasing the functional ability of the underactive dopaminergic system Preventing demyelization of the efferent cerebellar pathways

Increasing the functional ability of the underactive dopaminergic system.

In describing the ideal analgesic, what factors would be included? Select all that apply. Inexpensive Effective Have minimal adverse effects Decrease the level of consciousness Addictive

Inexpensive Have minimal adverse effects Effective Explanation: The ideal analgesic would be effective, nonaddictive, and inexpensive. In addition, it would produce minimal adverse effects and not affect the person's level of consciousness.

While assessing a critically ill client in the emergency department, the nurse notes on the cardiac monitor an R-on-T premature ventricular beat that develops into ventricular tachycardia (VT). Immediately, the client became unresponsive. The nurse knows that based on pathophysiologic principles, the most likely cause of the unresponsiveness is: massive cerebrovascular accident (CVA) resulting from increased perfusion. a blood clot coming from the heart and occluding the carotid arteries. interruption of the blood/oxygen supply to the brain. metabolic acidosis that occurs spontaneously following any dysrhythmias.

Interruption of the blood/oxygen supply to the brain. Explanation: The brain receives 15% to 20% of the total resting cardiac output and consumes 20% of its oxygen. The brain cannot store oxygen or engage in anaerobic metabolism. An interruption of blood supply or oxygen to the brain rapidly leads to clinically observable signs/symptoms. Unconsciousness occurs almost simultaneously with cardiac arrest. Metabolic acidosis will occur later in the cardiac arrest (but not immediately) and is not responsible for the client's unresponsiveness. CVAs can be caused by thrombosis formation or plaque occlusions, but is not the primary reason for unconsciousness in VT.

The instructor asks a group of nursing students to explain the function of the omentum. The students will respond based on which pathophysiologic principle? It holds organs in place. It attaches the jejunum and ileum to the abdominal wall. It has lots of mobility and moves around in the peritoneal cavity with peristaltic movements. It's mainly there to prevent any noxious substance from inner into the gut.

It has lots of mobility and moves around in the peritoneal cavity with peristaltic movements.

Feelings of dread, high anxiety, or exquisite pleasure can be elicited by stimulation of areas in which structure? Limbic system Cerebellum Temporal lobe Occipital lobe

Limbic system Explanation: Stimulation of specific areas of the limbic system can lead to feelings of dread, high anxiety, or exquisite pleasure. The temporal lobe is involved with auditory functions, the cerebellum with proprioception, and the occipital lobe with visual function.

When caring for the client with portal hypertension and ascites, which of these dietary interventions does the nurse suggest to prevent the progression of fluid accumulation? Avoid dairy products. Consume foods high in potassium. Reduce protein intake. Limit intake of sodium.

Limit intake of sodium.

What laboratory markers are most commonly used to diagnose acute pancreatitis? Cholesterol and triglycerides Lipase and triglycerides Amylase and cholesterol Lipase and amylase

Lipase and amylase

The nurse knows which phenomena listed below is an accurate statement about axonal transport? Axonal transport facilitates the movement of electrical impulses but precludes the transport of molecular materials. The unidirectional nature of the axonal transport system protects the CNS against potential pathogens. Materials can be transported to the nerve terminal by either fast or slow components. Anterograde and retrograde axonal transport allow for the communication of nerve impulses between and neuron and the central nervous system (CNS).

Materials can be transported to the nerve terminal by either fast or slow components. Explanation: The bidirectional axonal transport system allows for the transport of molecular materials (as opposed to electrical impulses); anterograde transport has both slow and fast components.

Which diagnostic finding is likely to result in the most serious brain insult? Mean arterial pressure (MAP) that equals intracranial pressure (ICP) High intracellular concentration of glutamate Increased ICP accompanied by hyperventilation Moderate decrease in brain tissue volume secondary to a brain tumor removal

Mean arterial pressure (MAP) that equals intracranial pressure (ICP). When the pressure in the cranial cavity approaches or exceeds the MAP, tissue perfusion becomes inadequate, cellular hypoxia results, and neuronal death may occur. Displacement of CSF and blood can partially compensate for decreased brain tissue volume. Hyperventilation partially mitigates, rather than exacerbates, increased ICP. Glutamate is normally in far higher concentrations intracellularly than extracellularly.

The nurse instructs the unlicensed assistive personnel (UAP) to be sure to turn the client every 2 hours in order to avoid pressure on the skin and avoid a pressure ulcer. What type of stimuli is the nurse encouraging the UAP to avoid? Thermal stimuli Pain stimuli Mechanical stimuli Chemical stimuli

Mechanical stimuli Explanation: Mechanical stimuli can arise from intense pressure applied to skin or from the violent contraction or extreme stretch of a muscle. Chemical stimuli arise from a number of sources, including tissue trauma, ischemia, and inflammation. Thermal stimuli can result from extremes of heat or cold. Pain stimuli is not a stimuli.

A client with a diagnosis of depression has been prescribed a medication that ultimately increases the levels of the neurotransmitter serotonin between neurons. Which process will accompany the actions of the neurotransmitter in a chemical synapse? More neurotransmitters will cross the synaptic cleft and bond with postsynaptic receptors. The neurotransmitter will cross gap junctions more readily. Communication between a neuron and the single neuron it is connected with will be facilitated. Two-way communication between neurons is permitted, in contrast to the one-way communication in electrical synapses.

More neurotransmitters will cross the synaptic cleft and bond with postsynaptic receptors. Explanation: In chemical synapses, neurotransmitters cross the synaptic cleft and bond with postsynaptic receptors to facilitate communication between neurons. This communication is one-way, not two-way, and each neuron has synaptic connections with thousands of other neurons. Gap junctions are associated with electrical synapses, not chemical synapses.

A nurse is teaching a client about the difference between digestion and absorption. Which action should the nurse describe as absorption? Emulsification of fats to make them water soluble Hydrolysis of compounds to make them into smaller molecules Moving nutrients from the external environment of the GI tract into the internal environment Cleavage of proteins into peptides and amino acids

Moving nutrients from the external environment of the GI tract into the internal environment

The nurse is caring for an elderly client with hemiplegia following a stroke. While planning the client's care, the nurse knows the client is at risk for developing which condition? Muscle atrophy Involuntary movements Pseudohypertrophy Muscular dystrophy

Muscle atrophy

Which of the following diseases is associated with fewer acetylcholine receptors, resulting in a lower-amplitude endplate potential, muscle weakness, and fatigability? Guillain-Barré syndrome Parkinson disease Muscular dystrophy Myasthenia gravis

Myasthenia gravis

Which is the primary component of white matter? Demyelinated lesions Cell bodies Dendrites Myelinated fibers

Myelinated fibers Explanation: Myelin has a high lipid content, which gives it a whitish color, and the name "white matter" is given to the masses of myelinated fibers in the spinal cord and brain. The other options are not myelinated.

During embryonic development, which structure develops into the central nervous system (CNS)? Neural tube Notochord Ectoderm Neural crest cells

Neural tube Explanation: During embryonic development, the neural tube develops into the CNS, whereas the notochord becomes the foundation around which the vertebral column ultimately develops. As the neural tube closes, ectoderm cells called neural crest cells migrate away from the dorsal surface of the neural tube to become progenitors of the neurons and supporting cells of the parasympathetic nervous system. The surface ectoderm separates from the neural tube and fuses over the top to become the outer layer of skin.

A client on an acute medicine unit of a hospital with a diagnosis of small bowel obstruction is reporting intense, diffuse pain in her abdomen. Which physiologic phenomenon is most likely contributing to her complaint? The client's C fibers are conducting pain in the absence of damaged Aδ (A-delta) fibers. First order neurons are inappropriately signaling pain to the dorsal root ganglion. Nociceptive afferents are conducting the sensation of pain along the cranial and spinal nerve pathways of the ANS. The client is experiencing neuropathic pain.

Nociceptive afferents are conducting the sensation of pain along the cranial and spinal nerve pathways of the ANS. Explanation: Visceral pain, as characterized by the client's description of her pain, is conducted by way of nociceptive afferents that use the cranial and spinal nerve pathways of the ANS. The problem is not likely rooted in the inappropriate firing of first-order neurons or the substitution of conduction by C fibers. Pain that is attributable to a pathological process apart from the neural pain network is not normally considered to be neuropathic.

A patient with a history of peptic ulcer disease presents to the emergency department with the following symptoms: early satiety, feeling of epigastric fullness and heaviness after meals, weight loss, and vomiting. The nurse suspects that the peptic ulcer has caused which of the following problems? Obstruction Obtrusion Penetration Perforation

Perforation

Multiple sclerosis is characterized by what type of neuron damage? Aneuropathy Transneuropathy Polyneuropathy Mononeuropathy

Polyneuropathy

Which treatment should take place immediately in a client experiencing autonomic dysreflexia? Apply binders and support hose to shunt the blood to the main organs. Increase the client's blood pressure to allow for adequate perfusion. Place the client in a supine position, and increase intravenous fluids. Position the client in upright position, and correct the initiating stimulus.

Position the client in upright position, and correct the initiating stimulus.

When educating a client about to undergo a pacemaker insertion, the nurse explains the normal phases of cardiac muscle tissue. During the repolarization phase, the nurse will stress that membranes must be repolarized before they can be re-excited. Within the cell, the nurse understands that: the cell membranes need to stay calm, resulting in muscle tissue becoming refractive. only the electrical activity within the heart will determine when repolarization occurs. the influx of calcium is the primary stimulus for the repolarization of cardiac tissue. potassium channels open while sodium channels close, causing repolarization to the resting state.

Potassium channels open and while sodium channels close causing repolarization to the resting state. Explanation: Repolarization is the phase during which the polarity of the resting membrane potential is re-established. This occurs with the closure of the sodium channels and opening of the potassium channels.

A nurse is completing an abdominal assessment on a patient suspected to have appendicitis. When the nurse applies and then releases pressure in the patient's right lower quadrant, the patient experiences tenderness. The nurse is documenting the presence of which of the following? Periumbilical tenderness Peforated appendix Referred tenderness Rebound tenderness

Rebound tenderness

Neurotransmitters exert their actions through specific proteins that are known as: Transformers Autoantigens Receptors Antibodies

Receptors Explanation: Neurotransmitters exert their actions through specific proteins that are known as receptors, embedded in the postsynaptic membrane.

A client is experiencing a cluster headache. The client would most likely manifest: nausea with vomiting. symptoms aggravated by physical activity. sensitivity to light. severe pain behind the eye.

Severe pain behind the eye Explanation: Symptoms of cluster headache include severe, unrelenting unilateral pain located in the orbital area. The pain radiates behind the eye to the ipsilateral trigeminal nerve. The client may also experience symptoms such as restlessness or agitation, conjunctival redness, lacrimation, nasal congestion, rhinorrhea, forehead and facial sweating, miosis, ptosis, and eyelid edema. The other options are associated with migraine headache.

A child is experiencing difficulty with chewing and swallowing. The nurse knows that which cells may be innervating specialized gut-related receptors that provide taste and smell? Special visceral afferent cells General visceral afferent neurons General somatic afferents Special somatic afferent fibers

Special visceral afferent cells Explanation: Special visceral afferent cells innervate specialized gut-related receptors. Their central processes communicate with special VIA column neurons that project to reflex circuits producing salivation, chewing, swallowing, and other responses. Special somatic afferent fibers are concerned with joint and tendon sensation. General somatic afferents respond to stimuli that produce pressure or pain. General visceral afferent neurons innervate visceral structures such as the GI tract, urinary bladder, and the heart and great vessels.

The health care provider is performing the withdrawal reflex test on a client. Which response would be expected? Stiffening of the extremity Moving away from the stimulus Straightening the limb Moving toward the stimulus

Straightening the limb Explanation: The withdrawal reflex is stimulated by a painful (nociceptive) stimulus and quickly moves the body part away from the offending stimulus, usually by flexing a limb part. The withdrawal reflex is powerful, taking precedence over other reflexes associated with locomotion.

The cause of gastric carcinomas has been influenced by which of the following factors? Select all that apply. Environmental factors Sexual history Family history Younger age Strain of Helicobacter pylori (H. pylori)

Strain of Helicobacter pylori (H. pylori) Family history Environmental factors

The nurse is explaining to a client's family how vasogenic brain edema occurs. The most appropriate information for the nurse to provide would be: The blood-brain barrier is disrupted, allowing fluid to escape into the extracellular fluid. There is an increase in the production of cerebrospinal fluid volume. There is a decrease in the amount of fluid volume in the brain. Normal physiologic circumstances result in decreased adsorption of CSF.

The blood-brain barrier is disrupted, allowing fluid to escape into the extracellular fluid. Vasogenic brain edema occurs with conditions that impair the function of the blood-brain barrier and allow the transfer of water and protein from the vascular space into the interstitial space. Increased production of CSF and decreased absorption result in hydrocephalus. It occurs in conditions such as hemorrhage, brain injury, and infectious processes. (less

During a clinical assessment of a 68-year-old client who has suffered a head injury, a neurologist suspects that a client has a sustained damage to her vagus nerve (CN X). Which assessment finding is most likely to lead the physician to this conclusion? The client is unable to turn her head from side to side and her tongue is flaccid. The client is unable to perform any fine motor movements of her tongue. The client has a unilateral facial droop, dry eyes, and decreased salivary production. The client has difficulty swallowing and has had recent constipation and hypoactive bowel sounds.

The client has difficulty swallowing and has had recent constipation and hypoactive bowel sounds. Explanation: Dysphagia and impaired GI motility are associated with damage to the vagus nerve. Lateral movement of the head is mediated by CN XI. Facial droop and dry eyes are associated with CN VII, the facial nerve, while abnormal tongue movement is a result of damage to CN XII, the hypoglossal nerve.

The nurse is performing a neurologic assessment on a client. Which assessment finding would the nurse determine is considered normal? Hesitates for 10 seconds and then able to blink eyes on command. The client cannot recognize a coin when placed in the hand but can recognize a cottonball. The client is unable to distinguish between two smells such as lemon and alcohol. The client is able to swing arms when walking.

The client is able to swing arms when walking. Explanation: The basal ganglia supply axial and proximal unlearned and learned postures and movements, which enhance and add gracefulness to UMN-controlled manipulative movements. These background movement functions are called associated movements. Intact and functional basal ganglia provide arm swinging during walking and running. Basal ganglia also are involved in follow-through movements that accompany throwing a ball or swinging a club.

An adult client has been admitted to a rehabilitation center after hospital treatment for an ischemic stroke. Which aspect of the client's history would be considered to have contributed to his stroke? The client takes corticosteroids for the treatment of rheumatoid arthritis. The client is an Asian male. The client's blood pressure has historically been in the range of 150/90 to 160/100 mm Hg. The man takes iron supplements for the treatment of chronic anemia.

The client's blood pressure has historically been in the range of 150/90 to 160/100 mm Hg. Explanation: Black ethnicity, male gender, hypertension, and diabetes are all well-documented risk factors for stroke. Anemia, autoimmune disorders like rheumatoid arthritis, and the use of corticosteroids are not noted to predispose to stroke.

A client has been recovering from a stroke for several weeks and has been reluctant to participate in physical therapy. As a result, the client has experienced disuse atrophy. The nurse should recognize that the client is experiencing the consequences of which physiologic process? The client's muscle cells have become denervated due to disuse The upper motor neurons controlling voluntary motor units have stopped firing A significant proportion of the client's muscle cells have died The diameter of the client's muscle cells has decreased

The diameter of the client's muscle cells has decreased

A client suffering a thrombotic stroke is brought into the emergency department by ambulance and the health care team is preparing to administer a synthetic tissue plasminogen activator for which purpose? Thrombolysis Hemolysis Thrombogenesis Hemostasis

Thrombolysis Synthetic tissue plasminogen activators work with the body's natural tissue plasminogen activators to convert plasminogen to plasmin, which breaks down clots to allow for the reestablishment of blood flow. There are two causes of strokes: hemorrhagic and thrombotic, with thrombotic strokes occurring much more frequently. Thrombolytics play a large role in increased outcomes seen with thrombotic strokes.

The most common cause of an ischemic stroke is which of the following? Thrombosis Vasospasm Intracerebral arterial vasculitis Cardiogenic embolus

Thrombosis Thrombi are the most common cause of ischemic strokes, usually occurring in atherosclerotic blood vessels.

The health care provider is assessing a client for carpal tunnel syndrome. The health care provider performs light percussion over the median nerve at the wrist. This assessment is known as: Chvostek sign Trousseau sign Phalen maneuver Tinel sign

Tinel sign

While reviewing the colonic absorption and role of flora in the GI system, the instructor will stress that the large intestine contains: a few species of anaerobic bacteria and no aerobic bacteria. a high percentage of acidic secretions that limit bacterial invasion. a complex microbial system that contains hundreds of different species. only a few species of bacterial species, primarily aerobic in nature.

a complex microbial system that contains hundreds of different species.

Which scenario would be an example of a child born with congenital insensitivity to pain? A child who: cries every time the wind blows because it hurts his face and ears. fell off a skate board and fractured ankle but did not feel any pain, just noted swelling in foot. skinned knee from a bike accident but only told parents when it started burning. develops pins-and-needles sensation after jumping out of a tree.

fell off a skate board and fractured ankle but did not feel any pain, just noted swelling in foot. Explanation: Analgesia is the absences of pain on noxious stimulation or the relief of pain without loss of consciousness. Congenital insensitivity is when a peripheral nerve defect apparently exists such that the transmission of painful nerve impulses does not result in perception of pain. Pins-and-needles sensation is called paresthesia. Burning sensations are usually associated with temperature (hyperthermia). Pain associated with wind (or any non-noxious stimuli) is called allodynia.


Kaugnay na mga set ng pag-aaral

Heart Attack, CPR, Cardiac Arrest

View Set

CompTIA A+ Certification Practice Test 16 (Exam 220-901)

View Set

What did you learn?? Heart Failure

View Set

General Biology II - Lecture 12 Quiz

View Set

TAX- Life/ Accident and Health Insurance + Life Polices

View Set

Microbiology Ch 21: Infectious Diseases Affecting the Genitourinary Systems

View Set